Riguroso argumento de bombeo de Laughlin

¿Hay alguna forma de justificar formalmente el argumento del bombeo de Laughlin? A menudo se argumenta que el flujo espectral de los niveles de Landau al variar el flujo insertado a través del anillo de Corbino debería dar cuenta de la conductividad cuantificada. En el calibre simétrico clasificamos los niveles de Landau con dos números cuánticos norte y metro . norte identifica el nivel, mientras que metro es básicamente el radio de la función de onda. La función de onda tiene efectivamente forma de anillo. Laughlin argumenta que tras la inserción de un flujo cuántico, los estados fluyen como:

ψ norte , metro ψ norte , metro + 1

Por lo tanto, un electrón por nivel de Landau lleno se mueve hacia afuera.

Ahora, el flujo espectral a menudo se introduce en el contexto del problema de una partícula en un anillo, donde tenemos un flujo de estados no degenerados. Aquí el teorema adiabático nos salvará de la mezcla de estados y después de la inserción de un cuanto de flujo, cada estado ganará una excitación, como si hubiéramos subido la escalera.

En el contexto del Quantum Hall efecto todos los metro Los estados son degenerados. Por lo tanto, la analogía con el caso anterior no se mantiene: no tenemos ningún argumento adiabático que proteja al estado mientras aumenta el flujo.

¿Existe una forma rigurosa de justificar la suposición de Laughlin?

Editar: en los comentarios se señaló que la partícula en un problema de anillo sufre una ambigüedad similar. Los niveles de energía cuando Φ es el flujo insertado son mi norte = 2 2 metro r 2 ( norte + Φ Φ 0 ) 2 . El espectro es doblemente degenerado cuando Φ Φ 0 = 1 2 . Invalidando así el argumento de la adiabaticidad. ¿Cómo podemos superar este problema?

Solo quiero señalar que existe una ambigüedad similar para el problema de una partícula en un anillo. Para la MAYORÍA de la evolución adiabática, los niveles de energía no son degenerados. Sin embargo, siempre hay un punto en el bombeo de flujo donde los niveles de energía se vuelven doblemente degenerados. Véase, por ejemplo, la mitad de la figura 2.1 aquí .

Respuestas (1)

Resolví el problema. Explicaré en detalle cómo la existencia de un operador conservado por la evolución adiabática nos permite dar sentido al argumento del flujo espectral. Comencemos con el problema de la partícula en un anillo. Aquí el hamiltoniano (para un flujo insertado estático Φ ) es:

H = 2 2 metro r 2 ( i ϕ + Φ Φ 0 ) 2

Siempre podemos diagonalizar ambos pag ϕ y H y obtenga una base completa del estado propio para el espacio de Hilbert de la partícula individual en un anillo. El estado propio en Φ = 0 son { mi i metro ϕ } . con energia mi norte = 2 2 metro r 2 metro 2 . Si insertamos un flujo dependiente del tiempo Φ ( t ) obtenemos:

H = 2 2 metro r 2 ( i ϕ + Φ ( t ) Φ 0 ) 2

El operador de cantidad de movimiento (hasta un ) es pag ϕ = i ϕ que se conserva por el encendido adiabático del flujo: [ H ( t ) , ϕ ] = 0 .

Si comenzamos la evolución con un estado propio ψ ( t = 0 ) del operador conservado:

pag ϕ ψ ( t = 0 ) = metro ψ ( t = 0 )
la evolución adiabática tu ( t ) conserva este estado:

tu ( t ) pag ϕ tu 1 ( t ) tu ( t ) ψ ( t = 0 ) = metro tu ( t ) ψ ( t = 0 )

pag ϕ ψ ( t ) = metro ψ ( t )

Así, incluso si por Φ Φ 0 = 1 2 hay una doble degeneración, los autoestados con diferentes metro Los valores no se mezclan. En otras palabras, después de la inserción adiabática del flujo, el número metro se conserva pero la energía ha aumentado: mi norte = 2 2 metro r 2 ( metro + 1 ) 2 . Básicamente el estado ψ metro se ha mudado a mi i ϕ ψ metro + 1 , donde la transformación de calibre se ha hecho explícita.

Ahora vemos el argumento del bombeo de Laughlin. Aquí el impulso pag ϕ se conserva por la evolución y, de hecho, en el calibre simétrico su valor propio es el número cuántico interno a cada nivel de Landau. Empezamos en un estado propio ψ metro del operador momento en el nivel más bajo de Landau. No hay mezcla con niveles más altos de Landau y el estado final debe tener un valor propio metro . El espectro del hamiltoniano después de la inserción del flujo se puede vincular al espectro del mismo antes de la inserción a través de una transformación de calibre: mi i ϕ ϕ metro . El vector propio con valor propio metro es mi i ϕ ϕ metro + 1 desde la exponencial mi i ϕ baje el valor propio en uno. El r dependencia de ϕ metro + 1 es r metro + 1 así, el estado se mueve hacia afuera después de una inserción adiabática, mientras mantiene su ϕ dependencia.

Estoy contigo hasta tu último párrafo, que parece ser un poco heurístico, y no puedo entender la lógica. De hecho, resumí mi confusión aquí ( physics.stackexchange.com/questions/650234/… ). Cualquier ayuda sería apreciada.
Y también estoy un poco confundido acerca de su transformación de calibre, ya que el calibre/ángulo ϕ no está bien definido cuando das la vuelta al círculo.